前言

今年這份考題難度適中,但一些題目至少綜合兩個數學觀念,學生必須對整體學習內容融會貫通才有機會取得高分。整體試題大致以由易至難的順序編排,單選題3題都很基本,多選題開始難度漸增,可以鑑別出中等及中上學生的程度。

難題的部份,多選5、6、8,題組13-14較難處理。尤其非選第14題,要運用積分概念估計積木體積的黎曼和,計算量較大,考生在有時間壓力的情況下不容易算對。

學習數學就像在搭鷹架,必須按部就班,長時間累積,才得以建立健全的觀念因應題目的變化,這有賴於平日學習習慣的培養。

相信會連到這個頁面的學生,至少已經是高二準備升高三的階段。我也相信,必然有用心的家長,為了孩子的學習在尋找資源,如果想要從根本改善學習數學的問題,必須從學習的本質出發去思考,為此,不妨參考以下這篇文章:如何學好高中數學?破除學習迷思,建立正確觀念

這個網站專注於製作與研發數位教材,目前已編寫 高一數學第1冊高一數學第2冊 免費提供給有需要的人使用。

完整試題及選擇(填)題參考答案下載

111年數甲試題111年數甲選擇(填)題參考解答

整體試題詳解與分析

單選1:等比數列與對數混合題

這一題只需使用「等比數列的定義」 與「對數基本運算」即可處理。
首先我們可以寫出此數列:\(a_1=10\)、\(a_2=10^2\)、\(a_3=10^3\)、\(a_4=10^4\)

因此
$$
\begin{aligned}
b &= log_{a_1}a_2+log_{a_2}a_3+log_{a_3}a_4 \\
&=log_{10}10^2+log_{10^2}10^3+log_{10^3}10^4 \\
&=2+\frac{3}{2}+\frac{4}{3} = 4\frac{5}{6}
\end{aligned}
$$

因此答案選(3)

單選2:三元一次方程組

首先,我們將此聯立方程組寫成矩陣的形式,接著以高斯消去法化簡
$$
\left[ \begin{array}{ccc|c}
1 & -1 & 1 & 0 \\
2 & c & 3 & 1 \\
3 & -3 & c & 0 \end{array} \right]
\rightarrow
\left[ \begin{array}{ccc|c}
1 & -1 & 1 & 0 \\
0 & c+2 & 1 & 1 \\
0 & 0 & c-3 & 0 \end{array} \right]
\rightarrow
\left[ \begin{array}{ccc|c}
1 & -1 & 1 & 0 \\
0 & c+2 & 0 & 1 \\
0 & 0 & c-3 & 0 \end{array} \right]
$$

化到這一步就可以判斷了。
我們從第二個矩陣可知,\(c-3\neq 0\),否則會有無限多組解。
接著化簡至第三個矩陣可知,當\(c+2=0\)時會造成無解。因此答案選(2)

單選3:空間坐標

依題意,可以假設\(P\)點座標為\(P(cos45^{\circ}, b, c)\),
接著看 \(P\) 點到 \(y\) 軸距離:
$$
\frac{\sqrt{6}}{3}=\sqrt{cos^2{45^{\circ}}+c^2}
$$
兩邊同時平方解方程式
$$
c^2=(\frac{\sqrt{6}}{3})^2-(\frac{\sqrt{2}}{2})^2 = \frac{6}{9}-\frac{1}{2} = \frac{1}{6}
$$
解得
$$
c=\pm\frac{\sqrt{6}}{6} (負不合)
$$

因此答案選(4)

多選4:多項式方程式

先以長除法計算 \(f(x)\)除以\(g(x)\)如下:

因為 \(f(x)\) 被 \(g(x)\) 整除,因此
$$a^2-2a-3=0, k-(2-a)=0$$
由 \(a^2-2a-3=0\)可解得 \(a=3\) 或 \(-1\)
若 \(a=3\),\(g(x)=x^2+3x+1=0\) 無虛根,與題意不合。
若 \(a=-1\),\(g(x)=x^2-x+1=0\) 有虛根,符合題意。

因此$$k-3=0 \ \ 解得\ \ k=3$$

接著來解方程式 $$f(x)=x^3+2x^2-2x+3=0$$
因為已經知道\(f(x)\)有一個因式\(g(x)=x^2-x+1\),所以 $$f(x)=x^3+2x^2-2x+3=(x^2-x+1)(x+3)$$
因此 \(f(x)=0\) 的三根為
$$
\frac{1\pm\sqrt{3}i}{2} \ \ or \ \ -3
$$

多選5:圓方程式

選項(1):令 \(y=0\),則 $$(x-1)^2+1^2=101$$
移項整理可解得 \(x=11\) 或 \(-9\),則 \(\Gamma\) 與 \(x\) 軸負向交於 \((-9,0)\)
另一方面,令 \(x=0\),則 $$1^2+(y-1)^2=101$$
移項整理可解得 \(y=11\) 或 \(-9\),因此 \(\Gamma\) 與 \(y\) 軸負向交於 \((0,-9)\)

選項(2):\(\Gamma\) 上 \(x\) 坐標最大的點是點 \((1+\sqrt{101}, 1)\)

選項(3):如以下圖所示,將原點與圓心 \((1,1)\) 相連,此直線與圓的兩個交點中,距離較遠的點即為此距離之最大值 \(\sqrt{2}+\sqrt{101}\)。

選項(4):此極坐標 \([9,\ theta]\) 表示與原點 \(O\) 距離皆為 9 的點,顯然 \(\Gamma\) 在第三象限的點與原點的距離並非固定為 9。故此選項不對。

選項(5): 令
$$
\left[\begin{array}{c}
x \\
y
\end{array}
\right]=
\left[\begin{array}{cc}
cos{\theta} & -sin{\theta} \\
sin{\theta} & cos{\theta}
\end{array}
\right]
\left[\begin{array}{c}
x’ \\
y’
\end{array}
\right]=
\left[\begin{array}{cc}
cos{\theta}x’-sin{\theta}y’ \\
sin{\theta}x’+cos{\theta}y’
\end{array}
\right]
$$

代入回方程式:
$$
\begin{aligned}
101 &= (x-1)^2+(y-1)^2 \\
&= x^2+y^2-2x-2y+2 \\
&= (cos{\theta}x’-sin{\theta}y’)^2+(sin{\theta}x’+cos{\theta}y’)^2 -2\cdot(cos{\theta}x’-sin{\theta}y’)-2\cdot(sin{\theta}x’+cos{\theta}y’)+2 \\
&= x’^2+y’^2-2\cdot(cos{\theta}+sin{\theta})x’+2(sin{\theta}-cos{\theta})y’+2
\end{aligned}
$$

經由以上計算可看出 \(xy\) 項係數為 \(0\),因此這個選項正確。

多選6:平面上的線性變換

依題意可知:
$$
\left[\begin{array}{cc}
a & b \\
c & d
\end{array}
\right]
\left[\begin{array}{cc}
1 & 0 \\
0 & 1
\end{array}
\right]=
\left[\begin{array}{cc}
3 & -\sqrt{3} \\
\sqrt{3} & 3
\end{array}
\right]
$$

選項(1):
$$
\left|\begin{array}{cc}
a & b \\
c & d
\end{array}
\right|
=
\left|\begin{array}{cc}
3 & -\sqrt{3} \\
\sqrt{3} & 3
\end{array}
\right|=9+3=12
$$

此選項不對。

選項(2):
$$
\left|\begin{array}{cc}
x’ \\
y’
\end{array}
\right]
=
\left[\begin{array}{cc}
3 & -\sqrt{3} \\
\sqrt{3} & 3
\end{array}
\right]
\left|\begin{array}{cc}
x \\
y
\end{array}
\right]
=
\left|\begin{array}{cc}
3x-\sqrt{3}y \\
\sqrt{3}x+3y
\end{array}
\right]
$$
$$
\begin{aligned}
\overline{OC’}^2 &=(3x-\sqrt{3}y)^2+(\sqrt{3}x+3y)^2 \\
&=12x^2+12y^2 \\
& = 12(x^2+y^2)=12\times 1 =12
\end{aligned}
$$
此選項正確

選項(3):設 \(\overrightarrow{OC}\) 和 \(\overrightarrow{OC’}\) 的夾角為 \(\theta\),則由內積的定義可知
$$
\begin{aligned}
cos{\theta}=\frac{\overrightarrow{OC}\cdot\overrightarrow{OC’}}{|\overrightarrow{OC}||\overrightarrow{OC’}|} &= \frac{x(3x-\sqrt{3}y)+y(\sqrt{3}x+3y)}{1\cdot 2\sqrt{3}} \\
&= \frac{3(x^2+y^2)}{1\cdot 2\sqrt{3}} = \frac{\sqrt{3}}{2}
\end{aligned}
$$
故 \(\theta=30^{\circ}\)

選項(4):已知 $$y’=\sqrt{3}x+3y$$
若 \(y=y’\),則 $$y=\sqrt{3}x+3y, 2y=-\sqrt{3}x$$

選項(5):
$$
\begin{aligned}
x’-y’&= (3x-\sqrt{3}y)-(\sqrt{3}x+3y) \\
&= (3-\sqrt{3})x-(\sqrt{3}+3)y \\
&= (3-\sqrt{3})[x-\frac{3+\sqrt{3}}{3-\sqrt{3}}] \\
&=(3-\sqrt{3})[x-(2+\sqrt{3})y]
\end{aligned}$$

由以上的式子找反例就容易多了:
必須符合
$$
\begin{cases}
x^2+y^2=1 \\
x<y \\
x>(2+\sqrt{3})y
\end{cases}
$$
取 $$x=-\frac{\sqrt{3}}{2},\ y=-\frac{1}{2}$$
即可。
因此選項(5)是錯的。

此題答案為 (2)、(4)

多選7:拋物線

我們來複習一下,拋物線的定義:拋物線上任一點到「焦點」及「準線」的距離是一樣的。因此
$$\overline{AA’}=\overline{AF}, \overline{BB’}=\overline{BF}$$

如下圖所示

通過 \(F\) 作一條鉛直線分別交直線 \(AA’\) 及 \(BB’\) 於 \(C\)、\(D\) 兩點。
另外,因為直線 \(FF’\) 與 \(BB’\) 平行,則 \(\angle{5}=\angle{F’BB’}\)。
接著,我們可以將比值 \(\frac{AF’}{AA’}\) 以下列三角函數表示:
$$
\begin{aligned}
\frac{\overline{AF’}}{\overline{AA’}} &= cot{\angle{1}} \\
&= tan{\angle{2}} \\
&= sin{\angle{3}} \\
&= sin{\angle{4}} \\
&=tan{\angle{5}}
\end{aligned}
$$

因此答案為 (3)、(5)

多選8:數列的極限

這一題出得不錯,
選項(1):由題意我們僅可以寫出以下不等式:$$b_n<a_n-\frac{4n-1}{n}$$
因此此選項不真。

選項(2):已知 $$ b_n+\frac{4n-1}{n}<3b_n $$
移項整理可得 $$ b_n>\frac{4n-1}{2n}$$
此項選正確。

選項(3):我們將題目給的不等式重新整理如下:
$$\frac{a_n}{3}<b_n<a_n-\frac{4n-1}{n}$$
接著計算極限:$$\lim_{n\rightarrow\infty}\frac{a_n}{3}=\lim_{n\rightarrow}(a_n-\frac{4n-1}{n})=2$$
由夾擠定理可知,$$\lim_{n\rightarrow\infty}b_n=2$$
即此數列收斂,故此選項不真。

選項(4)(5):$$a_n>b_n+\frac{4n-1}{n}>\frac{a_n}{3}+\frac{4n-1}{n}$$
將上式移項整理可得 $$a_n>\frac{3}{2}(4-\frac{1}{n})=6-\frac{3}{2}\cdot\frac{1}{n}$$
因此 $$a_{10000}>6-\frac{3}{2}\cdot\frac{1}{10000}=5.99985>5.9$$
故選項(4)錯誤,選項(5)正確。

最後答案應該選(2)(5)

選填9:期望值

首先我們先來計算得到紅包的機率
$$P=(\frac{1}{5})^2+\frac{4}{5}\cdot(\frac{1}{5})^2=\frac{9}{125}$$
接下來計算隨機變數 \(X\) 的期望值:
$$EX=1\cdot P+2(1-P)\cdot P+3(1-P)^2\cdot P+…$$
這是一個差比級數,計算方式就是將原式乘以 \(1-P\) 接著將兩式相減可得
$$P\cdot EX=P+(1-P)\cdot P+(1-P)^2\cdot P+…=\frac{P}{1-(1-P)}=1$$
因此 $$EX=\frac{1}{P}=\frac{125}{9}\approx 13.8 \approx 14$$

選填10:排列組合

因為英文的卷子不可在週二發出,那麼英文的卷子可能在下週一、三、四發出,我們依此分成三大類。
情況一:國文、英文均在週一,如下圖所示

此時,數、社、自三科排入下週二、三、四各一天,排法有 \(3!\) 種。

情況二:英文排在下週三,此時週一與週三皆有排科目了,因為每天至少排一科,所以剩下三科中,選兩科先排入下週二或四,剩下一科可能排在下週一、三或下週二、四。
如下圖所示,其排法有 \(P^3_2\times 2!\) 種。

另一種情況如下圖所示,其排法有 \(C^3_1\times 2\) 種。

最後一種情況則是,英文排在下週四,如下圖所示,排列數與英文排在下週三時相同。

最後,我們將以上情況加總
$$
\begin{aligned}
3!+(P^3_2\times 2!+C^3_1\times 2)\times 2
&= 6+(12+6)\times 2 \\
&= 6+36 = 42 (種)
\end{aligned}
$$

選填11:複數平面

首先我們要先解讀符號的意思:
在複數平面上,\(\frac{-3+4i}{5}\) 分別與 \(z^3\) 的距離與 \(z\) 的距離相等。且這三個點皆在單位圓上。可以先假設 \(z=cos{\theta}+isin{\theta}\),因此 \(z^3=cos{\theta}+isin{\theta}\)。
如下圖所示:

因此
$$
\begin{cases}
cos{2\theta}=-\frac{3}{5} — (1) \\
sin{2\theta}=\frac{4}{5} —(2)
\end{cases}
$$
由 (1) 式可知,$$2cos^2{\theta}-1=-\frac{3}{5}$$
可解得 $$cos{\theta}=\frac{\sqrt{5}}{5}, sin{\theta}=\frac{2\sqrt{5}}{5}$$

題組12-14 空間概念、積分

第12題

如圖所示,依題意標出數據如下:

同樣地,通過 \(D\) 點對平面 \(BCFE\) 作高,其垂足為 \(H\),如下圖所示:

由對稱性可知$$\overline{PM}=\frac{40-30}{2}=5$$
在 \(\Delta AMP\) 中,$$tan{\angle{AMP}}=\frac{15}{5}=3$$

第13題

同樣地,先依照題意標上符號與數據:

接著,將水平面 \(W\) 畫上去

以平行線截比例線段寫出 \(\overline{GH}\)
$$\frac{x}{15}=\frac{\overline{GH}}{10}, \overline{GH}=\frac{2}{3}x$$
另外
$$\frac{\overline{IJ}}{5}=\frac{x}{15}, \overline{IJ}=\frac{x}{3}$$
故此長方形的長=\(\overline{KL}=30+\frac{2}{3}x\),寬=\(\frac{2}{3}x\)
$$面積=(30+\frac{2}{3}x)\cdot(\frac{2}{3}x)=20x+\frac{4}{9}x^2$$
故得證。

第14題

依題意,將 \(\overline{AP}\)進行\(n\) 等份,每一等份的長度為 \(\frac{15}{n}\),長方形面積為 $$(30+\frac{10}{n}\cdot k)\cdot\frac{k}{n}\cdot 10$$

接著將每一塊長方體相加
$$
\begin{aligned}
& \sum_{k=1}^n(30+\frac{10}{n})\cdot\frac{k}{n}\cdot 10\cdot\frac{15}{n} \\
&= \sum_{k=1}^{n} \frac{4500}{n}\cdot\frac{k}{n}+\frac{1500}{n}(\frac{k}{n})^2 \\
&\rightarrow 4500\int_{0}^1 xdx + 1500\int_0^{1} x^2dx \\
&= 4500\cdot\frac{1}{2}+1500\cdot\frac{1}{3} \\
&= 2250 + 500 = 2750
\end{aligned}
$$

題組15-17 平面向量、微分

第15題

由題目的條件可知,我們必須先算出兩向量 \(\vec a \)、\(\vec b \) 的內積,再經由內積的定義寫出 \(cos{\theta}\)。此時可以先將 \(|\vec a+\vec b|=7\) 等式兩邊平方,再以 \(|\vec a|=x\),\(|\vec b|=9-x\)代入後,將\(cos{\theta}\)以 \(x\) 表示。
$$7^2=|\vec a -\vec b |^2=|\vec a| -2\vec a \cdot\vec b+|\vec b|^2$$

移項整理可得
$$
\begin{aligned}
cos{\theta} &= \frac{\vec a \cdot \vec b}{|\vec a| \cdot |\vec b|}\\
&= \frac{x^2-9x+16}{x(9-x)}\\
&= -1+\frac{16}{9x-x^2}
\end{aligned}
$$
此時 \(c=16, d=-1\)

令 \(f(x)=-1+\frac{16}{9x-x^2}, 1<x<8\)
$$
f'(x)=\frac{-16\cdot(9-2x)}{9x-x^2}=\frac{-144+32x}{(9x-x^2)^2}, 1<x<8
$$

第16題

函數 \(f(x)\) 的極值發生在臨界點:\(x=0, \ 9, \ \frac{9}{2}\)

可以先畫個表格觀察函數 \(f(x)\) 的遞增遞減區間:

\(f(x)\) 的最小值為
$$
f(\frac{9}{2})\geq -1+\frac{16}{\frac{81}{4}}=\frac{17}{81}
$$
即當 \(x=\frac{9}{2}\)時,\(\theta\)值最大。

第17題

函數 \(f(x)\) 的一次近似如下:
$$
\begin{aligned}
f(x) &\approx f(5)+f'(5)(x-5) \\
&= (-1+\frac{16}{20})+\frac{1}{25}(x-5) \\
&= -\frac{1}{5} +\frac{1}{25} (x-5)
\end{aligned}
$$
因此
$$
\begin{aligned}
cos{\theta}=f(4.96) &\approx -\frac{1}{5}+\frac{1}{25}(4.96-5) \\
&=-\frac{1}{5}+\frac{1}{25}(-0.04) \\
&=-\frac{1}{5}-\frac{0.04}{25}\\
&=-\frac{5.04}{25} = -\frac{126}{625}
\end{aligned}
$$

高中數學數位學習電子報

關於我